LSAT and Law School Admissions Forum

Get expert LSAT preparation and law school admissions advice from PowerScore Test Preparation.

 Paul Marsh
PowerScore Staff
  • PowerScore Staff
  • Posts: 290
  • Joined: Oct 15, 2019
|
#71290
Hi Carolyn!

So just from our first 2 rules, we can create this basic setup:

F :longline:
                G :longline: K
H :longline:

The 3rd rule is trickier to put in our diagram so let's come back to it. The fourth rule presents two scenarios: either M comes before H or after K. So now our setup follows two different possibilities.


Possibility 1:

F :longline:
                              G :longline: K
M :longline: H :longline:

Possibility 2:

F :longline:
               G :longline: K :longline: M
H :longline:


Ok, back to the 3rd rule, which tells us that if F is before M, then L is before H. Is F definitively before M in either of our possibilities? Yes, in possibility 2. So our final setup will look like this:

Possibility 1:

F :longline:
                              G :longline: K
M :longline: H :longline:


Possibility 2:

F :longline:
                              G :longline: K :longline: M
L :longline: H :longline:


In the second possibility, we have all the variables accounted for. In the first one, though, we still can't say where L goes. However, it can't quite go anywhere, since its placement is still limited by our second rule (for example: if we're in the first template, but we learn that F goes first, then we know L has to come before H and can therefore can only go second or third). So as we're answering the questions, the key is to figure out what template we're operating in. If we're in the second one, the order is accounted for by our diagram. If we're in the first one, we have to keep a close eye on where L goes.

Hope that helps!
 haganskl
  • Posts: 43
  • Joined: May 30, 2019
|
#75492
Hello. I would just like to check my understanding of rule #3.
If L is 6th is this the outcome from 1-6?
M F/H F/H G K L
Thanks in advance

Also, I understand from the discussion above that using the contrapositive of rule #3 would not have yielded all possibilities. My question is, would a third scenario of L-H yield all possibilities?
 Adam Tyson
PowerScore Staff
  • PowerScore Staff
  • Posts: 5153
  • Joined: Apr 14, 2011
|
#75496
Your proposed scenario with L 6th is valid, haganski! And it fits neatly into the first template that we described, the one based on M being before both H and K. In that template, L is random, so it can go anywhere in the sequence.

Often a conditional rule leads to three templates: 1) the sufficient occurs, the necessary occurs; 2) the necessary does not occur, the sufficient does not occur; and 3) the necessary occurs but the sufficient does not (in other words, the rule is simply not triggered). However, in this case there is no need for that additional template, because our first template includes that possibility. We based the templates not on the conditional third rule, but on the sequence rule four.

Good work!
 haganskl
  • Posts: 43
  • Joined: May 30, 2019
|
#75505
Thanks, Adam!

Yall are the best!
 startedfromKZnowwehere
  • Posts: 7
  • Joined: Jul 11, 2021
|
#89320
Hi there,

I really struggled with this game, but I can't seem to find others like it in the pure sequencing drill sets. Do y'all happen to know of any more sequencing games that have conditional elements (and so need multiple diagrams to contain all the possibilities) that I could practice with?

Thanks!
 Robert Carroll
PowerScore Staff
  • PowerScore Staff
  • Posts: 1787
  • Joined: Dec 06, 2013
|
#89336
startedfromKZnowwehere wrote: Tue Aug 03, 2021 9:29 pm Hi there,

I really struggled with this game, but I can't seem to find others like it in the pure sequencing drill sets. Do y'all happen to know of any more sequencing games that have conditional elements (and so need multiple diagrams to contain all the possibilities) that I could practice with?

Thanks!
I once made a list of these and similar games, so this would be a good place to start:

PT 51 Game 2 (hotel suites)
PT 61 Game 2 (artifacts)
PT 63 Game 2 (skydiving)
PT 63 Game 3 (vehicles)
PT 73 Game 1 (CD)
PT 74 Game 1 (band)
PT 78 Game 3 (antiques)
PT 83 Game 2 (concert)
PT 87 Game 1 (expert witness)

Hope this helps!

Robert Carroll
 startedfromKZnowwehere
  • Posts: 7
  • Joined: Jul 11, 2021
|
#89396
Robert Carroll wrote: Wed Aug 04, 2021 9:49 am
startedfromKZnowwehere wrote: Tue Aug 03, 2021 9:29 pm Hi there,

I really struggled with this game, but I can't seem to find others like it in the pure sequencing drill sets. Do y'all happen to know of any more sequencing games that have conditional elements (and so need multiple diagrams to contain all the possibilities) that I could practice with?

Thanks!
I once made a list of these and similar games, so this would be a good place to start:

PT 51 Game 2 (hotel suites)
PT 61 Game 2 (artifacts)
PT 63 Game 2 (skydiving)
PT 63 Game 3 (vehicles)
PT 73 Game 1 (CD)
PT 74 Game 1 (band)
PT 78 Game 3 (antiques)
PT 83 Game 2 (concert)
PT 87 Game 1 (expert witness)

Hope this helps!

Robert Carroll
Thanks so much! This is great.
User avatar
 amacdonald2121
  • Posts: 2
  • Joined: Jun 14, 2022
|
#102881
Hi! So the first time I did this problem I got the diagram and the inferences perfectly. Now I am reviewing and I am getting stuck on the inference where F-M and creates a complete chain. Does M have to come after K in this instance? The last rule says that either M - H or K - M. Is it possible that even if F -- M and L -- H that M can still come before H? I think I am missing something or overthinking this but some clarification would be much appreciated!
User avatar
 Jonathan Evans
PowerScore Staff
  • PowerScore Staff
  • Posts: 726
  • Joined: Jun 09, 2016
|
#102903
Hi, amacdonald!
Now I am reviewing and I am getting stuck on the inference where F-M and creates a complete chain. Does M have to come after K in this instance?
We have to start by looking at the two templates identified above.

The rules that govern the two templates are as follows:
  1. M before both K and H
  2. M after both K and H
F can come before M in both templates.

In template (2) F must be before M and M is right after K. M is definitely 6th here.

In template (1) F could be before M or F could be after M. If F is before M then this triggers the rule that L must be before H. If F is after M, nothing happens with L. In both scenarios in template (1) M is before K.
Is it possible that even if F -- M and L -- H that M can still come before H?
In template (1) it is possible for F to be before M, L to be before H, and M to be before H.

Get the most out of your LSAT Prep Plus subscription.

Analyze and track your performance with our Testing and Analytics Package.